2014-2015/2nd/DS/DS_0318/DS_0318.tex
2017-06-16 09:48:07 +03:00

337 lines
14 KiB
TeX

\documentclass[a4paper,10pt, table]{/media/documents/Cours/Prof/Enseignements/2014-2015/tools/style/classDS}
\usepackage{/media/documents/Cours/Prof/Enseignements/2014-2015/2014_2015}
% Title Page
\titre{6}
% \seconde \premiereS \PSTMG \TSTMG
\classe{\seconde}
\date{18 mars 015}
%\duree{1 heure}
%\sujet{%{{infos.subj%}}}
% DS DSCorr DM DMCorr Corr
\typedoc{DS}
\printanswers
\begin{document}
\maketitle
Le barème est donné à titre indicatif, il pourra être modifié.
\begin{questions}
\vfill
\question[4]
% developper
\begin{parts}
\part Relier les expressions égales entre elles. \textit{Il y a 4 liens à trouver. Un lien correct apporte 0,5points et un lien incorrect enlève 0,25points.}
\begin{minipage}[c]{0.5\textwidth}
\flushright
$ x(4x + 5) \qquad \bullet$ \\[0.5cm]
$ (-2-x)^2 \qquad \bullet$ \\[0.5cm]
$ (3x+1)^2 - 10 \qquad \bullet$ \\[0.5cm]
$ (x+1)(x-1) \qquad \bullet$ \\[0.5cm]
\end{minipage}
\hspace{2cm}
\begin{minipage}[c]{0.5\textwidth}
\begin{itemize}
\item $9x^2$
\item $x^2 - 1$
\item $-x^2 - 4x - 4$
\item $x^2 + 4x + 4$
\item $4x^2 + 5x$
\item $9x^2 + 6x - 9$
\item $2x^2 - 2$
\end{itemize}
\end{minipage}
\part Développer puis réduire les expressions suivantes
\begin{eqnarray*}
A = (3x + 4)(-2x + 1) & \qquad & B = (2x - 1)^2 + 2x
\end{eqnarray*}
\begin{solution}
\begin{eqnarray*}
A & = & (3x + 4)(-2x + 1) \\
A & = & 3x\times (-2x) + 3x \times 1 + 4 \times (-2x) + 4 \times 1 \\
A & = & -6x^2 + 3x - 8x + 4 \\
A & = & -6x^2 - 5x + 4
\end{eqnarray*}
\begin{eqnarray*}
B & = & (2x - 1)^2 + 2x \\
B & = & (2x - 1)(2x - 1) + 2x \\
B & = & 2x \times 2x + 2x \times (-1) + 2x\times (-1) + (-1) \times (-1) + 2x \\
B & = & 4x^2 - 2x - 2x + 1 + 2x \\
B & = & 4x^2 - 2x + 1
\end{eqnarray*}
\end{solution}
\end{parts}
\vfill
\question[6]
% statistiques
On veut étudier la répartition des salaires des ménages de Petit-Ville. Une étude statistique a été faite et a donné les résultats suivants:
\hspace{-1.5cm}
\begin{tabular}{|c|*{12}{c|}}
\hline
Salaires & 1300 & 1400 & 1500 & 1600 & 1700 & 1800 & 1900 & 2000 & 2100 & 2200 & 5300 & 8000 \\
\hline
Nombre de ménages & 13 & 9 & 9 & 7 & 16 & 12 & 10 & 8 & 8 & 3 & 4 & 1 \\
\hline
\end{tabular}
Des études similaires dans d'autres communes du département. Ces études ont donné les chiffres suivants
\begin{itemize}
\item Moyenne des salaires des ménages : 1800
\item Médiane des salaires des ménages : 1800
\end{itemize}
\medskip
\begin{parts}
\part
\begin{subparts}
\subpart Calculer avec la calculatrice la moyenne, les quartiles et la médiane de la série statistique conceranant Petit-Ville..
\begin{solution}
D'après la calculatrice, on lit
\begin{itemize}
\item Moyenne: 1906
\item $Q_1 = 1500$
\item $Me = 1700$
\item $Q_3 = 1900$
\end{itemize}
\end{solution}
\subpart Calculer l'étendue de cette série ainsi que l'espace interquartile.
\begin{solution}
Étendue de cette série
\begin{eqnarray*}
\mbox{Étendue} & = & max - min = 8000 - 1300 = 6700
\end{eqnarray*}
Espace interquartile:
\begin{eqnarray*}
\Delta Q & = & Q_3 - Q_1 = 1900 - 1500 = 400
\end{eqnarray*}
\end{solution}
\end{subparts}
\begin{EnvUplevel}
Le maire est interrogé par un journaliste au sujet de cette étude.
\begin{center}
\begin{itshape}
Ma ville est prospère! Je suis un bon maire, les habitants de Petit-Ville sont plus riches que les autres villes du département! Regardez notre salaire moyen est plus haut!
\end{itshape}
\end{center}
\end{EnvUplevel}
\part D'après votre étude êtes vous d'accord avec l'analyse du maire? Peut-on dire la même chose si on compare les médianes?
\begin{solution}
Si l'on compare les moyennes des revenus, oui le maire a raison le salaire moyen de Petit-Ville est supérieur à la moyenne des salaires des autres villes. Par contre si l'on compare les médiannes, la médiane de Petite-Ville est plus faible que la médiane des villes aux alentours.Donc 50\% de ceux qui gagnent le moins à Petite-Ville gagnent moins que les 50\% de ceux qui gagnent le moins dans les villes aux alentours.
\end{solution}
\part Pour expliquer l'écart entre la moyenne et la médiane, un journaliste décide de refaire cette étude en enlevant le foyer qui gagne 8000\euro.
\begin{subparts}
\subpart En enlevant le foyer qui gagne 8000\euro, calculer la moyenne, les quartiles et la médiane de la série statistique.
\begin{solution}
On enlève la valeur 8000 du tableau de la calculatrice et on obtient
\begin{itemize}
\item Moyenne: 1844
\item $Q_1 = 1500$
\item $Me = 1700$
\item $Q_3 = 1900$
\end{itemize}
Seul la moyenne a changé.
\end{solution}
\subpart Pensez vous que les habitants de Petit-Ville sont plus riches que les habitants des autres villes du département?
\begin{solution}
On remarque quand on a enlevé le foyer qui gagnait le plus, la moyenne des salaires est proche de la moyenne des villes autours. Les habitants ne sont pas plus riches que les habitants des alentours, il y a seulement un habitant qui gagne beaucoup d'argent.
\end{solution}
\end{subparts}
\end{parts}
\pagebreak
\question[5]
% vecteurs vue comme des forces
Une masse, représentée pas le point noir sur le dessin, est soumis à 2 forces: son poids (vecteur $\vec{P}$) et une aimantation (vecteur $\vec{A}$)
\begin{tikzpicture}[scale=0.7]
\draw[very thin, gray] (0,0) grid (10,10);
\coordinate (M) at (5,5);
\draw (M) node {$\bullet$};
\draw[->, very thick] (M) --++ (0,-3) node[midway, left] {$\vec{P}$};
\draw[->, very thick] (M) --++ (-3,4) node[midway, above right] {$\vec{A}$};
\ifprintanswers
\draw[->, very thick, red] (M) --++ (0,-3) --++ (-3,4) node[midway, below] {$\vec{A}$};
\draw[->, very thick, red] (M) --++ (-3,1) node[midway, above] {$\vec{A} + \vec{P}$};
\draw[->, very thick, blue] (M) --++ (3,-1) node[midway, above] {$\vec{F}$};
\fi
\end{tikzpicture}
\begin{parts}
\part Lire sur le dessin les coordonnées des vecteurs $\vec{P}$ et $\vec{A}$.
\begin{solution}
D'après le dessin on compte
\begin{eqnarray*}
\vec{P} = \vectCoord{0}{-3} & \qquad & \vec{A} = \vectCoord{-3}{4}
\end{eqnarray*}
\end{solution}
\part
\begin{subparts}
\subpart Tracer sur le dessin le vecteur $\vec{P} + \vec{A}$.
\subpart Calculer la somme $\vec{P} + \vec{A}$.
\begin{solution}
Calculons cette somme
\begin{eqnarray*}
\vec{P} + \vec{A} & = & \vectCoord{0}{-3} + \vectCoord{-3}{4} = \vectCoord{0 + (-3)}{-3 + 4} = \vectCoord{-3}{1}
\end{eqnarray*}
\end{solution}
\subpart Est-ce que la masse bouge?
\begin{solution}
Comme la somme des forces appliquées à la masse n'est pas nulle, la masse bouge.
\end{solution}
\end{subparts}
\part Déterminer les coordonnées de la force supplémentaire $\vec{F}$ à appliquer pour que la masse ne bouge pas.
\begin{solution}
Pour que la masse ne bouge pas, il faut que la somme des forces soit nulle. Il faut donc \textit{contrer} la somme des forces $\vec{A}$ et$\vec{P}$. Si l'on choisit $\vec{F} = \vectCoord{3}{-1}$ (en bleu sur le dessin), on peut vérifier que la somme des 3 vecteurs est nulle:
\begin{eqnarray*}
\vec{F} + \vec{A} + \vec{P} & = & \vectCoord{3 - 3 + 0}{-1 + 4 - 3} = \vectCoord{0}{0} = \vec{0}
\end{eqnarray*}
Donc la masse est en équilibre, elle ne bouge pas.
\end{solution}
\end{parts}
\vfill
\question[5]
% vecteurs vue comme des déplacement
\begin{tikzpicture}[scale=0.7]
\repere{-6}{6}{-5}{5}
\ifprintanswers
\coordinate (A) at (-3,1);
\coordinate (B) at (0,3);
\coordinate (C) at (-2,-1);
\coordinate (D) at (-5,-3);
\coordinate (E) at (3,-1);
\draw (A) node {$\bullet$} node[below] {$A$};
\draw (B) node {$\bullet$} node[right] {$B$};
\draw (C) node {$\bullet$} node[below] {$C$};
\draw (D) node {$\bullet$} node[below] {$D$};
\draw (E) node {$\bullet$} node[below] {$E$};
\draw[->, very thick] (D) -- (A) node[midway, above left] {$\vec{DA}$};
\draw[->, very thick] (C) -- (B) node[midway, above left] {$\vec{CB}$};
\draw[->, very thick, blue] (E) --++ (2,4) node[midway, above left] {$\vec{DA}$} node[above] {$F$};
\draw[->, very thick, red] (A) --++ (7,4) node[midway, above ] {$\vec{AG}$} node[above] {$G$};
\draw[->, very thick, red] (A) --++ (3,-4) node[midway, above right] {$\vec{BE}$} --++ (4,8) node[midway, above left] {$2\vec{CB}$};
\fi
\end{tikzpicture}
Soit $A(-4;1)$, $B(0;3)$, $C(-2;-1)$, $D(-5;-3)$ et $E(3;-1)$.
\\[0.3cm]
\begin{parts}
\part Placer les points sur l'annexe.
\begin{solution}
Voir sujet
\end{solution}
\part Calculer les coordonnées des vecteurs $\vec{DA}$, $\vec{CB}$.
\begin{solution}
Coordonnées du vecteur $\vec{DA}$:
\begin{eqnarray*}
\vec{DA} & = & \vectCoord{x_A - x_D}{y_A - y_D} = \vectCoord{-3 - (-5)}{1 - (-3)} = \vectCoord{2}{4}
\end{eqnarray*}
Coordonnées du vecteur $\vec{CB}$:
\begin{eqnarray*}
\vec{CB} & = & \vectCoord{x_B - x_C}{y_B - y_C} = \vectCoord{0 - (-2)}{3 - (-1)} = \vectCoord{2}{4}
\end{eqnarray*}
\end{solution}
\part Quelle est la nature du quadrilatère $DABC$. Justifier.
\begin{solution}
D'après la question précédente, les vecteurs $\vec{DA}$ et $\vec{CB}$ ont les mêmes coordonnées donc le $\vec{DA} = \vec{CB}$ donc le quadrilatère $DABC$ est un parallelogramme.
\end{solution}
\part On note $F$ le point tel que $\vec{EF} = \vec{DA}$.
\begin{subparts}
\subpart Placer le point $F$ sur le dessin.
\subpart Calculer les coordonnées du point $F$.
\begin{solution}
D'après la question 1, on sait que $\vec{DA} = \vectCoord{2}{4}$. De plus $\vec{EF} = \vectCoord{x_F - x_E}{y_F - y_E} = \vectCoord{x_F - 3}{y_F - (-1)}$ donc
\begin{eqnarray*}
\vectCoord{x_F - 3}{y_F + 1} = \vectCoord{2}{4}
\end{eqnarray*}
La première coordonnée donne:
\begin{eqnarray*}
x_F - 3 = 2 & \equiv & x_F = 2 + 3 = 5
\end{eqnarray*}
Et la deuxième coordonnée donne
\begin{eqnarray*}
y_F + 1 = 4 & \equiv & y_F = 4 - 1 = 3
\end{eqnarray*}
Donc $F$ a pour coordonnées $(5;3)$, et on peut vérifier que l'on ne s'est pas trompé avec le dessin.
\end{solution}
\end{subparts}
\part On note $G$ le point tel que $\vec{AG} = \vec{BE} + 2\vec{CB}$.
\begin{subparts}
\subpart Placer le point $G$ sur le dessin.
\subpart Calculer les coordonnées du point $G$.
\begin{solution}
On commence par calculer les coordonnées des vecteurs suivants
\begin{eqnarray*}
\vec{BE} & = & \vectCoord{x_E - x_B}{y_E - y_B} = \vectCoord{3 - 0}{-1 -3} = \vectCoord{3}{-4} \\
\vec{CB} & = & \vectCoord{x_B - x_C}{y_B - y_C} = \vectCoord{0 - (-2)}{3 - (-1)} = \vectCoord{2}{4} \\
2\vec{CB} & = & \vectCoord{2 \times 2}{2 \times 4} = \vectCoord{4}{8} \\
\vec{BE} + 2\vec{CB} & = & \vectCoord{3 + 4}{-4 + 8} = \vectCoord{7}{4} \\
\vec{AG} & = & \vectCoord{x_G - x_A}{y_G - y_A} = \vectCoord{x_G - (-3)}{y_G - 1} = \vectCoord{x_G + 3}{y_G - 1}
\end{eqnarray*}
Donc on en déduit que
\begin{eqnarray*}
\vectCoord{x_G + 3}{y_G - 1} & = & \vectCoord{7}{4}
\end{eqnarray*}
Donc la première ligne donne
\begin{eqnarray*}
x_G + 3 = 7 & \equiv & x_G = 7 - 3 = 4
\end{eqnarray*}
Et la deuxième ligne
\begin{eqnarray*}
y_G - 1 = 4 & = & y_G = 4 + 1 = 5
\end{eqnarray*}
Donc les coordonnées de $G$ sont (4, 5).
\end{solution}
\end{subparts}
\end{parts}
\vfill
\end{questions}
\end{document}
%%% Local Variables:
%%% mode: latex
%%% TeX-master: "master"
%%% End: